GMAT Manhattan Prep Verbal

¡Supera tus tareas y exámenes ahora con Quizwiz!

In a certain country, insurance companies pay only a fixed percentage - set by law and uniform across all insurance companies - of the fees actually billed by healthcare providers; the remaining balance is waived. Patients without insurance coverage, however, are legally responsible for the full amount of those fees. If the fixed percentage set for insurance companies is changed, healthcare providers will adjust their fees so that the insurance companies pay the same dollar amount that they would have paid before the change. The information given most strongly supports which of the following general claims about the country described? a) If the percentage prescribed by law for insurance companies is decreased, private patients without insurance coverage will pay more for healthcare services. b) The dollar amount paid by insurance companies to healthcare providers will not change unless the legally specified percentage is changed. c) If the number of patients without insurance coverage grows beyond the number of patients covered by insurance, healthcare providers will adjust their billing practices according to the needs of those patients. d) If an economic recession causes the healthcare industry to suffer, the legally prescribed percentage paid by insurance companies will be increased. e) Acquiring insurance coverage, if possible, would be a sound economic decision for any patient.

(1) Identify the Question Type At first this may seem to be a Strengthen question, because of the phrase "most strongly supports." However, we are asked which answer choice is supported by the information in the passage, so this is an Inference question. (2) Deconstruct the Argument Under the law, insurance companies only have to pay a fixed percentage of healthcare bills, but private patients must pay the full amount. In addition, if the percentage is changed, healthcare providers will adjust the amount of the bill, so that the dollar amount does not change. If the legal percentage is decreased, healthcare providers must increase the billing level in order to offset the change. Alternatively, if the legal percentage is increased, healthcare providers would have to decrease the billing level in order to offset the change. (3) State the Goal On inference questions, we're looking for the answer that must be true according to the information given in the passage. The most tempting wrong answers are often items that might be true but don't have to be true. (4) Work from Wrong to Right (A) CORRECT. The passage states that, if the percentage is changed, healthcare providers will adjust the amount of the bill so that the dollar amount does not change. In particular, if the legal percentage is decreased, healthcare providers must bill increased amounts to accomplish this objective. Since private patients are responsible for the full amount billed, they will be responsible for the new, higher amount. (B) There is nothing in the passage to suggest that the amount paid by insurance companies could not change for reasons unrelated to the law. For example, nothing in the passage prevents the healthcare providers from changing their prices, and a change in prices without a change in the percentage paid would result in a change in the dollar amount paid by the insurance companies. (C) This statement might be true, but there is nothing in the passage to indicate that it must be true. The passage describes what healthcare providers will do only in the event of a change in the law; it does not describe what they will do in other situations, such as the one described in this answer choice. (D) The passage provides no information about what determines the legally set percentage. (E) This statement may be true for some patients but does not have to be true for all, e.g., for those who use healthcare services very infrequently. For such patients - even if they pay greater bills when they do use healthcare services - the difference may not make up for the cost of insurance premiums, especially if those premiums are high.

A group of experimental subjects participated in an "intermittent fasting" study, under which they ate all of their food for the day within six hours of waking up. The subjects consumed the same number of calories as they normally did throughout an entire day and did not change their exercise patterns. Nearly all of the subjects lost a significant amount of weight during the study. It can thus be concluded that eating all of one's food within a relatively short period of time causes the body to burn more calories. Which of the following would be most useful to establish in evaluating the argument? a) Are people more likely to consume low-calorie foods early in the day than at night? b) Is the practice of intermittent fasting safe and free of major side effects? c) Are most people able to consume as many calories within an interval of six hours as they normally would over the course of an entire day? d) Will people following an intermittent fasting protocol feel substantially hungrier than those who space out their meals more regularly? e) Does the body burn calories faster when food is eaten earlier in a person's waking hours than when it is eaten later?

(A) If this were true, perhaps the subjects would have consumed fewer calories, but the premise states that calorie consumption remained the same, so this is out of scope. (B) The conclusion is only about the effect of eating in short periods on calorie consumption. The argument doesn't make any claims about safety, nor does it advocate the practice of intermittent fasting, so it isn't necessary to determine whether this practice is safe to evaluate the argument. (C) Even if most people are unable to fit a full day's calories into six hours, the subjects in the study did just that. The argument is about the effect of eating all one's calories over a short period, not whether intermittent fasting could be adopted more widely without calorie reduction, so it's not necessary to make this determination. (D) The argument is only concerned with the effects of timing on calorie burning, so hunger is out of scope. (E) CORRECT. This could provide an alternative explanation for the study result. Perhaps people burned more calories simply because they ate earlier in the day and not because of the condensed time period. If this wouldn't happen in the evening, perhaps the author's conclusion is not true in general. For instance, perhaps consuming all one's calories in the last six hours of the day would lead to weight gain, not loss. In that case, it wouldn't appear that simply eating over a reduced period of time lead to increased calorie burning.

Many of today's mathematicians use computers to test cases that [are either too time-consuming or involve too many variables to test manually, allowing the exploration of] theoretical issues that were impossible to test a generation ago. (A) are either too time-consuming or involve too many variables to test manually, allowing the exploration of (B) either take too much time or involve too many variables to be tested manually; allowing the mathematicians to explore (C) would either take too much time or involve too many variables to test manually, allowing them to explore (D) would either be too time-consuming or would involve too many variables to test manually; this capability allows the mathematicians to explore (E) take too much time or variables to test manually; this capability allows the mathematicians to explore

According to this sentence, many of today's mathematicians use computerized testing for two kinds of cases: first, those that take too much time for manual testing, and, second, those containing too many variables for manual testing. The structure used to introduce these cases, either X or Y, demands parallelism. (A) Too time-consuming and involve too many variables are not parallel. Because the verb are appears before the parallelism marker either, parallelism dictates the following reading of the second item: to test cases that are involve too many variables. The second item has two verbs! (B) A semi-colon (;) is used to connect two complete sentences. In this choice, the second portion (allowing...) is not a complete sentence. (C) CORRECT. Take too much time and involve too many variables are properly parallel. The change to the hypothetical form would is acceptable because the sentence indicates that these cases would not be tested if they had to be tested manually. The pronoun them is also acceptable, as it points back to the subject of the first half of the sentence (many of today's mathematicians use computers, allowing them to explore...). (D) Be too time-consuming and would involve too many variables are not parallel. Because the verb would appears before the parallelism marker either, parallelism dictates the following reading of the second item: to test cases that would would involve too many variables. The second item repeats the word would! (E) The word variables is a countable noun, so it needs to be modified by the adjective many, not the adjective much (which is used for uncountable nouns). This choice may seem more concise than others, but it introduces a modifier error.

According to Italy's top anti-Mafia prosecutor, [the ailing mobster came to take refuge in Corleone, a town famous because of "The Godfather" and near to those he most trusted.] (A) the ailing mobster came to take refuge in Corleone, a town famous because of "The Godfather" and near to those he most trusted (B) famous because of "The Godfather," the ailing mobster came to take refuge in Corleone, a town near to those he most trusted (C) the ailing mobster, famous because of "The Godfather," came to take refuge in Corleone, a town near to those he most trusted (D) near to those he most trusted, the ailing mobster came to take refuge in Corleone, a town famous because of "The Godfather" (E) Corleone, famous because of "The Godfather," was the town that the ailing mobster came to take refuge in because it was near to those he most trusted

Answer: A The original sentence is correct. "Famous because of 'The Godfather'"and "near to those he most trusted" correctly modify "a town," which modifies Corleone. Noun modifiers must be next to the nouns that they describe. This choice contains no other errors. (A) CORRECT. This answer choice is correct as it repeats the original sentence. (B) This choice contains a modification error; "famous because of 'The Godfather'" incorrectly describes the prosecutor. Noun modifiers modify the closest available noun. (C) This choice contains a modification error; "famous because of 'The Godfather'" incorrectly describes the mobster. Noun modifiers modify the closest available noun. (D) This choice contains a modification error; "near to those he most trusted" incorrectly describes the prosecutor. Noun modifiers modify the closest available noun. (E) The modification is correct in this choice. "Famous because of 'The Godfather'" correctly modifies "Corleone". However, this sentence is unnecessarily wordy, "was the town that the ailing mobster came to take refuge in" is much less concise than "the ailing mobster came to take refuge in Corleone" without making the meaning clearer.

Although there has been great scientific debate for decades over global warming, most scientists now agree that human activity is causing the Earth's temperature to rise. Though predictions vary, many global warming experts believe that average global temperatures will rise between three and eight degrees Fahrenheit during the next century. Such an increase would cause an alarming rise in sea levels, displacing millions of people by destroying major population centers along the world's coastlines. Which of the following is an assumption in support of the argument's conclusion? a) New technological developments in the next century will not divert rising seas from the world's coastal cities. b) Individuals will not become more aware of the steps they can take to reduce the emission of greenhouse gases. c) Rising sea levels similarly affect all coastal population centers. d) Some global warming experts predict a greater than eight degree Fahrenheit increase in global temperatures during the next century. e) Human activity is the sole cause of increasing global temperatures.

Answer: A The argument concludes that rising sea levels caused by global warming will destroy major coastal population centers and displace millions of people. Any assumption in support of this conclusion would have to corroborate that these events will definitively take place. (A) CORRECT. If new technological developments in the next century allow people to divert rising seas from the world's cities (i.e., population centers), cities will not be destroyed and millions of people will not be displaced. Thus, a necessary assumption is that these technologies will not be developed. (B) A simple awareness of the steps to reduce emissions in no way undermines the argument's conclusion, as this answer choice does not describe any action being taken by individuals. Additionally, greenhouse gases are never mentioned as the primary by-product of human activity that causes global warming, and are therefore not sufficient to address the argument. (C) The argument never suggests that all coastal population centers are similarly affected; this choice is too extreme and overreaching for the argument's conclusion. (D) This might be true, but it is not an assumption on which the conclusion rests. Instead, this answer choice is simply an inference that might be drawn from the premises. (E) The idea that human activity is the sole cause of global warming is neither suggested nor assumed by the argument. In addition, the wording "sole cause" is too extreme.

[Galileo did not invent the telescope, but on hearing, in 1609, that such an optical instrument had been made, he] quickly built his own device from an organ pipe and spectacle lenses. A. Galileo did not invent the telescope, but on hearing, in 1609, that such an optical instrument had been made, he B. Galileo had not invented the telescope, but when he heard, in 1609, of such an optical instrument having been made, C. Galileo, even though he had not invented the telescope, on hearing, in 1609, that such an optical instrument had been made, he D. Even though Galileo did not invent the telescope, on hearing, in 1609, that such an optical instrument had been made, E. Even though Galileo did not invent the telescope, but when he heard, 1609, of such an optical instrument being made, he

Answer: A http://gmatclub.com/forum/galileo-did-not-invent-the-telescope-but-on-hearing-in-111155.html

When drive-ins were at the height of their popularity in the late 1950s, some 4,000 existed in the United States, but today [there are less than one-quarter that many] A. there are less than one-quarter that many B. there are fewer than one-quarter as many C. there are fewer than one-quarter of that amount D. the number is less than one-quarter the amount E. it is less than one-quarter of that amount

Answer: B tl;dr "amount" and "less" are for uncountable things http://gmatclub.com/forum/when-drive-ins-were-at-the-height-of-their-popularity-in-the-44864.html

[Among the sports played primarily by women in the United States is field hockey, which is one of] the most popular men's sports in South Asia, and volleyball, the sport of many male athletes in Europe and South America. a) Among the sports played primarily by women in the United States is field hockey, which is one of b) In the United States, among the sports played primarily by women are field hockey, one of c) Played in the United States primarily by women, field hockey is among d) Sports played primarily by women in the United States, including field hockey, one of e) One of the sports played in the United States primarily by women is field hockey, among

Answer: B This sentence is intended to highlight a contrast: in the United States, field hockey and volleyball are primarily women's sports, but, in other areas of the world, they are heavily contested by men. To convey this message effectively, the sentence must employ several grammatical elements properly. First, modifiers must be carefully placed. For instance, the construction played primarily by women in the United States, present in the original prompt, suggests the interpretation that women in the United States constitute most of the participants in these sports—an illogical interpretation, given the immediately following facts. The sentence must therefore be rearranged to clarify the idea that in the United States these are primarily women's sports, but elsewhere they are not. Second, field hockey... and volleyball is a compound noun. Therefore, if this noun is the subject of a verb in the sentence, that verb must be plural. Finally, field hockey and volleyball are both followed by descriptive modifiers; ideally, those modifiers should be expressed in forms that are as similar as possible. (A) The singular verb is does not agree with the plural noun field hockey... and volleyball. Additionally, the modifiers describing field hockey and volleyball (which is one of... and the sport of...) are not expressed in similar forms. Finally, the modifier in the United States appears to describe women, thus illogically implying that American women make up most of the world's field hockey and volleyball players. (B) CORRECT. In this sentence, the modifier In the United States is clearly placed so as to modify the entire following clause, clarifying the idea that the status of field hockey and volleyball as women's sports is a phenomenon limited to the United States (and not illogically implying that American women constitute most of the world's field hockey and volleyball players). The plural subject field hockey... and volleyball is correctly paired with the plural verb are. Finally, the modifiers following field hockey and volleyball are both appositive modifiers, strengthening the cohesiveness of that parallel structure. (C) This choice creates the complete clause field hockey is among the most popular men's sports in South Asia. As a result, the element following "and"—consisting only of volleyball and a modifier—is not parallel to anything, and the resulting construction is a run-on sentence. In addition, the modifier Played in the United States primarily by women, which is intended to describe both sports, appears to describe only field hockey. (D) The modifier in the United States appears to describe women, thus illogically implying that American women make up most of the world's field hockey and volleyball players. Additionally, this choice creates a sentence fragment. The noun sports is followed only by modifiers; there is no main verb. (E) The singular verb is does not agree with the plural noun field hockey... and volleyball. Additionally, this choice illogically describes field hockey and volleyball as "one of the sports...", but these are in fact two different sports.

In some African languages, verbs [not only encode the timeframe of an event but also imply the origin of the speaker's knowledge, which may be direct observation, hearsay, or intuition, resulting in speakers of those languages who cannot state facts without an attribution to some source.] (A) not only encode the timeframe of an event but also imply the origin of the speaker's knowledge, which may be direct observation, hearsay, or intuition, resulting in speakers of those languages who cannot state facts without an attribution to some source (B) not only encode the timeframe of an event but also the origin of the speaker's knowledge, direct observation, hearsay, or intuition; therefore, speakers of those languages cannot state a fact without some source of attribution (C) encode not only the timeframe of an event but also the origin of the speaker's knowledge, whether direct observation, hearsay, or intuition; as a result, speakers of those languages cannot state facts without attributing them to a source (D) do not encode the timeframe of an event; they also imply the origin of the speaker's knowledge -- whether direct observation, hearsay, or intuition -- resulting in the inability of those languages' speakers to state facts and not attributions to some source (E) not only encode the timeframe of an event but also imply the origin of the speaker's knowledge, direct observation, hearsay, or intuition; speakers of those languages, therefore, do not state facts without attributing them to sources

Answer: C According to the sentence, verbs in certain African languages serve two functions: first, they place events into certain timeframes, and, second, they imply the source of the speaker's knowledge of those events. These functions should be placed in parallel, with a construction that emphasizes the additional functionality of these verbs. The three potential sources of the speaker's knowledge should also appear in a clearly demarcated parallel structure. Finally, the sentence should accurately describe the result—namely, the fact that speakers of these languages cannot describe facts alone, but must instead attribute those facts to some source. (A) It is unclear exactly what is described by the modifier resulting.... Moreover, the modifier itself does not make sense: the phrasing resulting in speakers... seems to suggest that the existence of the speakers themselves, rather than their inability to state facts without attributions, is the result of the linguistics described in the sentence. (B) This version lacks proper parallelism: the parallel marker not (only) is followed by a verb (encode...), but its counterpart but (also) is followed by a noun (the origin...). The three potential sources of the speaker's knowledge do not appear in a clearly demarcated parallel structure; instead, that parallel structure seems, illogically, to contain four items (the origin, direct observation, hearsay, and intuition). Finally, the wording source of attribution has the wrong meaning; it appears to refer to the source of the attribution, rather than to the source of the fact itself. (C) CORRECT. The two types of information encoded by the verbs are properly expressed in parallel (not only the timeframe ... but also the origin). In addition, the parallel construction whether x, y, or z is correctly used to list the three potential sources of the speaker's knowledge. This sentence uses a semicolon to separate two parts that are themselves complete sentences (independent clauses). (D) The first clause incorrectly implies that the verbs actually do not encode the timeframe of an event. Not only is this meaning illogical in context, but the transition also cannot logically be used to connect a negative statement and a positive statement. In addition, the parallel structure facts and not attributions illogically suggests that facts and attributions are two independent entities. (E) The three potential sources of the speaker's knowledge do not appear in a clearly demarcated parallel structure; instead, that parallel structure seems, illogically, to contain four items (the origin, direct observation, hearsay, and intuition). In addition, the verb phrase do not state... does not properly convey the idea that the speakers actually cannot state facts without attributions; it merely suggests that they do not do so, by habit or custom.

The media claim that the economy is entering a phase of growth and prosperity. They point to lower unemployment rates and increased productivity. This analysis is false, though. The number of people filing for bankruptcy has increased every month for the last six months, and bankruptcy lawyers report that they are busier than they have been in years. a) Unemployment rates are not useful indicators of growth and prosperity. b) Economic growth cannot be measured in terms of productivity. c) Legislation has not been recently passed to make legal bankruptcy easier to obtain. d) There has not been an increase in the number of bankruptcy lawyers. e) The media often misrepresent the current state of economic affairs.

Answer: C The conclusion of the argument is that the media are wrong in saying that the economy is entering a phase of growth and prosperity. The basis for that claim is that the number of people filing for bankruptcy has increased every month for the last six months and that bankruptcy lawyers are busier than they have been in years. In order for this argument to be valid, however, the author has to assume that the increase in the number of bankruptcies is a result of the state of the economy and not the result of something unrelated. (A) This statement does not have to be true for the claim that the media are wrong about the economy to hold. Even if unemployment rates are useful indicators of growth and prosperity, the media could still be wrong about the economy (e.g., if there are other indicators that show problems in other areas). (B) This does not have to be true for the conclusion to hold. Productivity could be a good measure of economic growth, but the media could still be wrong about the economy (e.g., if there are other indicators that show problems in other areas). (C) CORRECT. This has to be true for the conclusion to hold. If legislation has recently been passed that makes it easier to obtain bankruptcy, then the evidence cited would be less relevant. The increased number of bankruptcies could have been the result of the easier process rather than of a poor economy. (D) This does not have to be true for the conclusion to hold. An increase in the number of bankruptcy lawyers would not explain the increase in the number of bankruptcy filings. (E) This does not have to be true for the claim that the media are wrong about the economy to hold. Even if the media did not often misrepresent the current state of economic affairs, the argument that the media are wrong might still hold.

According to a survey of graduating medical students conducted by the Association of American Medical Colleges, [minority graduates are nearly four times more likely than are other graduates in planning to practice] in socioeconomically deprived areas. (A) minority graduates are nearly four times more likely than are other graduates in planning to practice (B) minority graduates are nearly four times more likely than other graduates who plan on practicing (C) minority graduates are nearly four times as likely as other graduates to plan on practicing (D) it is nearly four times more likely that minority graduates rather than other graduates will plan to practice (E) it is nearly four times as likely for minority graduates than other graduates to plan to practice

Answer: C http://gmatclub.com/forum/according-to-a-survey-of-graduating-medical-students-80793.html

For the last five years, the XYZ Courier Company has made regular delivery trips between Town A and Town B. The average time taken by the company's drivers to drive the round trip between the two towns, excluding the time taken for loading, unloading, and delivery, over that period has been 80 minutes. John, a driver for XYZ, needs to make a personal trip between the two towns; he figures that he should allow approximately 80 minutes for the round trip. Which of the following, if true, does not call John's conclusion into question? a) The route between Town A and Town B has been plagued by increasing congestion over the last five years, as the area's population has doubled during that time. b) Most of XYZ's courier vehicles are heavy trucks, for which speed limits are lower than for passenger vehicles. c) Many of the packages carried by XYZ between Town A and Town B are large, high-security packages, for which the processes of loading, unloading, and delivery can take up to half the length of the trip itself. d) John will make his personal trip at an hour when XYZ does not make delivery trips. e) Before a freeway was built between Town A and Town B two years ago, the only routes between the two towns were state highways with multiple traffic lights and reduced-speed downtown zones.

Answer: C John assumes that his round-trip driving time between the two towns will be similar to XYZ's average time, over the last five years, for the same round trip. Therefore, John's conclusion will be weakened by (a) any systematic difference between John's personal trip and XYZ's company trips, or (b) any reason why the five-year average is not representative of the time currently required to drive between the two towns. (A) If this statement is true, then the five-year average commute time will not be representative of the current commute time, since the increasing congestion will have raised the current commute time to a level greater than the five-year average. (B) If most of XYZ's courier vehicles are subject to speed limits lower than those for commuter automobiles, then the courier vehicles' commute time will probably be longer than John's commute time in his personal vehicle. (C) Correct The cited round-trip time specifically excludes the time required for loading, unloading, and delivery, so the length of time taken by these tasks is irrelevant to the conclusion at hand. (D) Since traffic patterns are highly dependent upon the time of day, the given 80-minute average cannot necessarily be expected to hold at times when XYZ does not make deliveries. (E) If this statement is true, then, for the first three years of the five-year period in question, the transit time between the two towns would have been much longer than that for the last two years of the period (and thus the current time). Therefore, in this case, the five-year average will not be representative of the current time.

Joanne: An increasing number of online retailers now allow customers to create "wish lists" of items they would like to receive as gifts. Such lists are certainly useful, but these retailers should also explore other ways to suggest gift purchases for these customers. In particular, without revealing the specifics of a customer's purchase history, a website could quickly analyze a retailer's entire inventory, select a list of items similar to those the customer has already purchased, and then e-mail that list to a group of contacts specified by the customer. Such a system would suggest gifts that, because of their similarity to the customer's prior purchases, would be extremely likely to appeal to the customer. In the argument, Joanne assumes that the hypothetical customers (A) are familiar with most or all of the items in stock on the websites where they shop (B) would prefer novel gifts that are unlike the items they currently own (C) do not use retail websites primarily to purchase gifts (D) would be relatively unconcerned if their retail purchase histories were available to others (E) prefer online shopping to shopping in physical retail stores

Answer: D (1) Identify the Question Type The problem asks what is assumed by Joanne, so this is a Find Assumption question. (2) Deconstruct the Argument Joanne sees the introduction of "wish lists" on retail websites as a positive development. As an additional way to suggest gifts for customers, she says, retail websites should analyze those customers' previous purchases and generate gift ideas that are similar to those purchases. Because these items would be similar to things that the customer has already bought, Joanne reasons, they would be ideal gifts. (3) State the Goal We need to find what is taken for granted by Joanne in the argument. Her fundamental point is that a list of items similar to the customer's prior purchases should accurately reflect things that the customer would like for himself or herself (and would therefore make good gift ideas). To justify the premise that a customer's purchase history should reflect what that customer wants for himself or herself, Joanne must assume that the purchase history does not consist mostly of items bought for other people. It must also assume that a customer wants things similar to what they already own. If a person bought a toaster, does that necessarily mean that they want another toaster? As there are a few assumptions made, we will have to see how the answer choices are phrased before making a decision. (4) Work from Wrong to Right (A) Whether the customer is familiar with most of a retailer's inventory is irrelevant to Joanne's suggestion. Her proposed system would scan a retailer's entire catalog, whether the target customer is familiar with most of the products or not. (B) In fact, Joanne assumes precisely the opposite: Her system is based on the idea that things similar to what a customer already owns will make good gifts. (C) CORRECT: For the argument to work, Joanne must assume that customers use retail websites primarily to purchase things for themselves. More specifically, Joanne must assume that customers do not use these websites primarily to purchase items for other people, such as gifts or resale items. Therefore, this statement is an assumption. We can also use the negation method. If this statement is false, then the hypothetical customer uses retail websites primarily to purchase gifts for other people. In that case, the customer's purchase history will reflect the desires of the people to whom the customer gives gifts, rather than those of the customer him- or herself -- thus destroying the effectiveness of Joanne's argument. (D) Joanne's proposed system would not reveal customers' purchase histories, so this consideration is irrelevant. (E) While Joanne does assume that customers have built up a purchase history with online retailers, she makes no assumptions about their preferences between online shopping and shopping in stores.

The subway system in New City has recently been running a severe budget deficit. Analysts have concluded that, to avoid a crisis, subway revenues must increase by 50% from the current level, beginning with the coming quarter. Subway riders are currently charged a flat fare per ride; the chairperson of New City's transit authority has concluded that a 50% increase in this fare, effective at the beginning of the coming quarter, will be sufficient to avert any crisis. Each of the following, if true, calls into question the chairperson's conclusion EXCEPT a) Subway fares are paid with single-ride tokens that are purchased in advance and do not expire. b) On the most heavily traveled routes in New City's downtown, express bus fares range from 25 to 40 percent greater than current subway fares. c) Under New City's tax code, subway fare increases of more than 25 percent trigger reductions in the amount of tax money allocated to the subway system. d) New City's economy is adding many more jobs in suburban areas, which are inaccessible by subway, than in the urban areas that the subway system serves. e) Mobile phones, upon which a significant percentage of New City's workers have come to depend for essential communication while commuting, will be banned on the subway from the coming quarter onward.

Answer: D (1) Identify the Question Type The language "calls into question" and "if true" indicates that this is a Weaken the Conclusion question. The "EXCEPT" variation indicates that we need to find and eliminate 4 "weaken" answers. The "odd one out" answer, an answer that does not weaken, will be the correct answer. (2) Deconstruct the Argument In order to avert a budget crisis, New City's transit authority must increase subway revenues by 50 percent. The chair of the transit authority has devised a plan to increase subway fares by exactly this percentage and believes that this will avert any crisis. In other words, the chair is assuming that a 50 percent fare increase will lead directly to a 50 percent increase in actual revenues and that this increase will occur as soon as the plan is implemented. (3) State the Goal We're asked to find something that does not weaken the argument. The four (incorrect) answers, which will weaken the argument, must each make the conclusion at least somewhat less likely to be true or valid. The correct answer will either strengthen the conclusion or be irrelevant to the conclusion. Implicit in the stated plan is the assumption that nothing will offset the fare increase or delay an immediate increase in revenues. The chairperson assumes that ridership levels will not drop and that there won't be any other reasons why the 50% fare increase might not result in an immediate revenue increase of 50%. (4) Work From Wrong To Right (A) If prepaid tokens will remain valid, current subway riders who purchase a large number of tokens at current prices will be able to avoid paying the increased fare for some time - thus delaying the projected increase in revenue. The chairperson assumes that the fare increase will produce the desired result more or less immediately, an assumption that is undermined by this choice. (B) If this statement is true, then the proposed subway fare increase will raise subway fares to a level higher than the corresponding express bus fares. In that case, many current subway riders will have an incentive to switch from the subway to the express buses. If some do switch, revenues are likely to decrease. (C) The proposed subway fare increase is far greater than 25 percent, so, if this statement is true, the increase will trigger a reduction in the tax revenue given to the subway system. That tax reduction will offset the revenue increase from the greater fare, thus undermining the chairperson's plan. (D) CORRECT. The greater rate of job growth in areas outside the reach of the subway does not, by itself, contain or imply any basis for a negative effect on subway revenues. In particular, there is no reason to assume that the number of workers who ride the subway will decrease. In fact, the wording of this choice suggests that New City is actually adding jobs in areas served by the subway, albeit more slowly than in the suburbs; thus, if anything, the subway's ridership is more likely to increase (however slowly) than to decrease. The chairperson's argument requires only that the ridership not decrease, so this choice does not weaken that argument. (E) This choice states that "a significant percentage" of New City's professionals have come to view mobile phones as essential, specifically during commute times. If phones are banned, many current subway riders will have an incentive to switch from the subway to another mode of transportation; if so, revenues are likely to decrease.

Transfer [molding is a process in which polymer resins are superheated, compressed in a charger, and then they are injected into a closed mold cavity; it is used to produce plastic moldings with variable wall thicknesses, as well as fine surface detail, such as the exterior panels of buses and trolleys. ] a) molding is a process in which polymer resins are superheated, compressed in a charger, and then they are injected into a closed mold cavity; it is used to produce plastic moldings with variable wall thicknesses, as well as fine surface detail, such as the exterior panels of buses and trolleys b) molding—a process in which superheated polymer resins are compressed in a charger, then injecting into a closed mold cavity—is used to produce plastic moldings, such as the exterior panels of buses and trolleys with variable wall thicknesses, as well as fine surface detail c) molding—a process in which superheated polymer resins compressed in a charger are then injected into a closed mold cavity—it is used to produce plastic moldings, such as the exterior panels of buses and trolleys, with walls of variable thickness and surfaces of fine detail d) molding, a process in which superheated polymer resins are compressed in a charger and then injected into a closed mold cavity, is used to produce plastic moldings, such as the exterior panels of buses and trolleys, with variable wall thicknesses and fine surface detail e) molding is a process involving the compression of superheated polymer resins in a charger, and then they are injected into a closed mold cavity; this process, which is used to produce plastic moldings with variable wall thicknesses and fine surface detail, such as the exterior panels of buses and trolleys

Answer: D (A) The sentence presents a list of three things: superheated, compressed, and then they are injected. The third item is not parallel to the first two. The placement of the modifier such as the exterior panels of buses and trolleys illogically suggests that those panels are examples of fine surface detail, rather than examples of plastic moldings in general. (B) The use of injecting is illogical, suggesting that the resins themselves actually inject some other chemical into the cavity; injected should be used instead, as the resins themselves are injected into the cavity. At the end of this choice, such as the exterior panels of buses and trolleys with variable wall thicknesses is written as a single example; if the example is cut out, the remaining part of the sentence (is used to produce plastic moldings as well as fine surface detail) is illogical. (C) The core of the sentence contains two subjects without any conjunction between: Transfer molding it is used to produce. This is not an acceptable sentence. (D) CORRECT. The core structure (Transfer molding is used to produce) is a proper sentence. The modifier such as the exterior panels correctly refers to plastic moldings, and the two features of those moldings (variable wall thicknesses and fine surface detail) are properly expressed in parallel. (E) The construction and they are injected can only be considered parallel to transfer molding is a process. In a parallel construction using a pronoun for the second subject, the first expectation is that the pronoun they refers to the original subject, transfer molding. It does not; the pronoun they logically refers to resins. Further, the portion following the semicolon is a sentence fragment, consisting only of a noun (this process) and modifiers.

[Based on records from ancient Athens, each year young Athenian women collaborated to weave a new woolen robe that they used to dress] a statue of the goddess Athena and that this robe depicted scenes of a battle between Zeus, Athena's father, and giants. A. Same B. Based on records from ancient Athens, young Athenian women had collaborated to weave a new woolen robe with which to dress C. According to records from ancient Athens, each year young Athenian women collaborated to weave a new woolen robe that they used to dress D. Records from ancient Athens indicate that each year young Athenian women collaborated to weave a new woolen robe with which they dressed E. Records from ancient Athens indicate each year young Athenian women had collaborated to weave a new woolen robe for dressing

Answer: D http://gmatclub.com/forum/based-on-records-from-ancient-athens-each-year-young-135912.html

Most of the purported health benefits of tea [comes from antioxidants""compounds also found in beta carotene, vitamin E, and vitamin C that] inhibit the formation of plaque along the body's blood vessels. A. comes from antioxidants""compounds also found in beta carotene, vitamin E, and vitamin C that B. comes from antioxidants""compounds that are also found in beta carotene, vitamin E, and vitamin C, and they C. come from antioxidants""compounds also found in beta carotene, vitamin E, and vitamin C, and D. come from antioxidants""compounds that are also found in beta carotene, vitamin E, and vitamin C and that E. come from antioxidants""compounds also found in beta carotene, vitamin E, and vitamin C, and they

Answer: D https://www.manhattanprep.com/gmat/forums/most-of-the-purported-health-benefits-of-tea-t2558.html

[As the former] chair of the planning board for 18 consecutive years and a board member for 28 years, Joan Philkill attended more than 400 meetings and reviewed more than 700 rezoning applications. A. As the former B. The former C. Former D. She was E. As the

Answer: E http://gmatclub.com/forum/as-the-former-chair-of-the-planning-board-for-18-consecutive-69993.html

[Stymieing the Armada's plans to meet up with the Duke of Parma's army off the coast of Flanders, the reason for the defeat of the Spanish Armada was not only due to gale winds that favored the British but also the sacrificing] of eight war ships as "fire ships," vessels filled with flammable materials and sent downwind toward the closely-anchored Spanish fleet. (A) Stymieing the Armada's plans to meet up with the Duke of Parma's army off the coast of Flanders, the reason for the defeat of the Spanish Armada was not only due to gale winds that favored the British but also the sacrificing (B) The defeat of the Spanish Armada, which stymied the Armada's plans to meet up with the Duke of Parma's army off the coast of Flanders, was not only due to gale winds that favored the British but also the sacrifice (C) The defeat of the Spanish Armada, which stymied the Armada's plans to meet up with the Duke of Parma's army off the coast of Flanders, was not only due to gale winds that favored the British but also the sacrificing (D) Stymieing the Armada's plans to meet up with the Duke of Parma's army off the coast of Flanders, the reason for the defeat of the Spanish Armada was not only gale winds that favored the British but also the sacrificing (E) Stymieing the Armada's plans to meet up with the Duke of Parma's army off the coast of Flanders, the defeat of the Spanish Armada was due not only to gale winds that favored the British but also to the sacrifice

Answer: E The original sentence begins with an opening modifier: Stymieing the Armada's plans... Flanders. The subject following the modifier should be whatever stymied the Armada's plans, but the reason did not stymie its plans. Rather, the defeat of the Armada prevented it from later meeting up with the Duke of Parma's army. (A) The opening modifier incorrectly modifies the reason. It is also redundant to write the reason for X was due to Y. Only one of the two (reason or due to) is needed. (B) The X and Y elements in the idiom not only X but also Y must be parallel, but due to gale winds is not parallel to the sacrifice. Either due to must be placed before not only (due to not only gale winds but also the sacrifice) or it must be repeated for both elements (not only due to gale winds but also due to the sacrifice). (C) The X and Y elements in the idiom not only X but also Y must be parallel, but due to gale winds is not parallel to the sacrificing. Either due to must be placed before not only (due to not only gale winds but also the sacrificing) or it must be repeated for both elements (not only due to gale winds but also due to the sacrificing). (D) The opening modifier incorrectly modifies the reason. (E) Correct. The opening modifier correctly modifies the defeat. The X and Y elements in due not only to gale winds but also to the sacrifice are parallel.

A recent research study of undergraduate students analyzed the effects of music on human emotions. Each of the 200 participants attended at least 1 two-hour concert of classical music per week over the course of 12 weeks of their spring semester. At the end of the experiment, all of the students filled out a questionnaire assessing their emotional state. Based on the results of the questionnaires, all of the 10 students who attended the greatest number of concerts reported lower stress levels and higher satisfaction with their lives. Also, most of the 20 students who attended the fewest number of concerts reported below-average levels of emotional comfort. Which of the following must be true based on the evidence presented above? a) Most of the 200 participants improved their emotional state and lowered their stress levels. b) During each week of the experiment, the participants spent at least 2 hours less on their academic work as a result of concert attendance. c) Listening to classical music for at least 2 hours per week improves the emotional well-being of the majority of young adults. d) More than 6 participants attended at least 14 concerts during the course of the experiment. e) At least some of the students participated in the study in order to gain free access to classical concerts.

The argument above provides a detailed description of a research study. Note that the question stem contains only factual information and does not have a conclusion. Our task is to provide the most probable conclusion for this argument, i.e. the one that must be true based on the provided evidence. When looking for the most probable conclusion, remember not to make any additional assumptions and choose the answer that directly follows from premises stated in the argument. (A) This statement does not have to be true. The argument provides evidence about the emotional progress of only 30 participants. The fact that we have no information about the vast majority of participants demonstrates that the statement in this answer choice cannot be justified. (B) While each participant did spend at least 2 hours each week attending the concerts, there is no information in the argument that would suggest that the students reduced their study time. For example, they could have attended the concerts in their free time. (C) This statement does not have to be true, since we have no information about the emotional progress of the vast majority of study participants. Note that even if the study did demonstrate a positive effect of classical music on the majority of participants, it would still be uncertain whether this effect would hold for the majority of young adults. (D) CORRECT. We know that 20 students attended the fewest number of concerts, 10 students attended the greatest number of concerts, and the remaining 170 students attended some other number of concerts in between. The term 'greatest' indicates that there are at least 3 different numbers of concerts attended by the students (as opposed to 'greater' to distinguish between 2 different numbers). Since each of the participants attended at least one concert per week during the 12 weeks of the experiment, all of the study participants must have attended at least 12 concerts. Even if the 20 bottom students attended the smallest possible number of concerts (i.e. 12), it must be the case that the next 170 students in the middle attended at least one more (i.e. at least 13 concerts) and the 10 most active participants must have attended at least one more than the middle group, i.e at least 14 concerts. Thus, it must be true that the 10 most active participants (i.e. more than 6 participants) attended at least 14 concerts, as stated in this answer choice. Note that if the students attended more concerts than the minimum requirement, the number of students with at least 14 concerts attended will be even greater, still validating the accuracy of this statement. (E) The argument does not explicitly state whether the participants received free access to the concerts or had to pay for admission (e.g. they could have just received a discount). In addition, no information is provided about the motivation of study participants.

When housing prices climb too quickly, the Federal Reserve often responds by raising the key interest rate, [which has the curious effect of actually lowering housing prices instead of raise them], as one might expect. a) which has the curious effect of actually lowering housing prices instead of raise them b) an action that has the curious effect of actually lowering housing prices rather than raising them c) which has the curious effect of actually lowering housing prices instead of raising them d) which rather than raising housing prices actually has the curious effect of lowering them e) an action that has the curious effect of actually lowering housing prices instead of raise them

In the original sentence, the use of "which" incorrectly implies that the key interest rate has the curious effect, when in fact it is the raising of the interest rate that does. Also, "lowering housing prices instead of raise them" is not a parallel construction. And it is preferable to use "rather than" with verbs, in place of "instead of," which is better used with nouns. (A) This choice is the same as the original sentence. (B) CORRECT. This choice remedies the incorrect use of "which." Moreover, it contains the parallel construction "lowering housing prices rather than raising them." Finally, "rather than" is correctly used here in place of "instead of." (C) This choice corrects the parallelism issue, but it still incorrectly uses "which" and "instead of." (D) This choice incorrectly uses "which." (E) This choice incorrectly uses "instead of" and contains the unparallel construction "lowering housing prices instead of raise them."

The health commissioner said that the government had implemented strict measures to eradicate the contaminated food and, despite the recent illnesses, [it will try] to prevent the outbreak from recurring in the future. a) it will try b) that it tried c) it had tried d) it would have tried e) that it would try

In the original sentence, the verb "had implemented" is in the past perfect tense, indicating that this event occurred at some point before the commissioner spoke. The verb "will try", however, is in the simple future. When the future is indicated from the point of view of the past, the simple future is not used. Instead, the conditional is required. For example, "The man said that he would buy a new car" is preferable to "The man said that he will buy a new car." We need to find a conditional verb. Moreover, the pronoun "it" begins a new clause and thus requires repetition of "that" in order to make clear, using parallel structure, that this new clause is still something that the commissioner said. For example, "The man said that he would buy a new car and that he would drive it everywhere" is preferable to "The man said that he would buy a new car and he would drive it everywhere." (A) This choice is incorrect as it repeats the original sentence. (B) This choice does not offer the conditional "would try", though it does offer another "that". The past tense "tried" is definitely wrong here because the trying will happen "in the future" according to the original sentence. Thus this choice changes the meaning unacceptably. (C) This choice uses the past perfect tense "had tried" where the conditional "would try" is preferred. An extra "that" is needed to make the two clauses "the government had..." and "it had tried" parallel. (D) This is a tempting choice as it fixes the verb tense to the conditional "would." However, the tense is technically "conditional perfect" (would have tried), which is not the proper tense. Moreover, an extra "that" is needed to make the two clauses "the government had..." and "it would try" parallel. (E) CORRECT. This choice provides the plain conditional tense and another "that".

[The physicist Richard Feynman presented a comprehensive introduction to modern physics designed for undergraduate students in a two-year course.] a) The physicist Richard Feynman presented a comprehensive introduction to modern physics designed for undergraduate students in a two-year course. b) For undergraduate students, the physicist Richard Feynman presented a two-year course, being a comprehensive introduction to modern physics. c) A comprehensive introduction was in a two-year course by the physicist Richard Feynman presenting to undergraduate students an introduction to modern physics. d) Presenting a comprehensive introduction, the physicist Richard Feynman introduced modern physics in a two-year course designed for undergraduate students. e) In a two-year course designed for undergraduate students, the physicist Richard Feynman presented a comprehensive introduction to modern physics.

Modifiers: The original sentence suggests that Feynman's introduction covered "physics designed for undergraduate students." This is nonsensical; rather the course is designed for undergraduate students and covers the general topic of physics. (A) This choice is incorrect as it repeats the original sentence. (B) Beginning the sentence with "for undergraduate students" is awkward and unclear. The verb phrase "being a comprehensive introduction..." following the comma seems illogically to modify "the physicist Richard Feynman." With the use of the unnecessary "being," this creates the awkward suggestion that "the physicist" was "a comprehensive introduction." (Remember that "being" is almost always wrong on the GMAT.) (C) The sentence's meaning is unclear due to the use of many prepositional phrases with no punctuation: "In a two-year course" followed by "by the physicist Richard Feynman" and later, "to undergraduate students" and "to modern physics." Also, the subject of this passive sentence is "a comprehensive introduction." It would make more sense for Feynman to be the subject, since he was actively doing something: "presenting." Finally, the use of "presenting" with the passive construction introduces a verb tense error; Feynman is not currently "presenting" the course, rather, the course was presented by Feynman. (D) The use of both "introduction" and "introduced" is redundant: it suggests that Feynman "introduced a comprehensive introduction." (E) CORRECT. The placement of the prepositional phrase "in a two-year course designed for undergraduate students" at the beginning of the sentence clarifies the meaning: a physics course was designed for the students. The construction of the rest of the sentence is straightforward: the subject (the person doing the action) "the physicist Richard Feynman," the verb (what he actually did) "presented," and the object (what he presented) "a comprehensive introduction to modern physics."

If, in a tennis tournament, a match reaches a fifth-set tiebreak, the lower-ranked player always loses the tiebreak (and, therefore, the match). If Rafael, the second-ranked player, wins a tournament by beating Roger, the top-ranked player, then the match must not have included a fifth-set tiebreak. Which of the following arguments most closely mimics the reasoning used in the above argument? a) If a woman with a family history of twins gets pregnant three times, she will have one set of twins. Jennifer, who falls into this category, had two sets of twins, so she must not have gotten pregnant exactly three times. b) If a salesman sells more product than anyone else in a calendar year, then he will earn an all-expenses-paid vacation. Joe earned an all-expense-paid vacation, so he must have sold more product than anyone else for the year. c) A newspaper can charge a 50% premium for ads if its circulation surpasses 100,000; if the circulation does not pass 100,000, therefore, the newspaper can't charge any kind of premium for ads. d) If a student is in the top 10% of her class, she will earn a college scholarship. Anna is not in the top 10% of her class, so she will not earn a scholarship. e) All of the players on a football team receive a cash bonus if the team wins the Super Bowl. If quarterback Tom Brady earned a cash bonus last year, he must have been a member of the winning Super Bowl team.

On a "mimic the argument" question, it's useful to use logic notation to understand the flow of the argument. In this case, we're told that IF A happens (a match reaches a fifth-set tiebreak), THEN B will definitely happen (the lower-ranked player loses). Standard logic rules tell us that, when given "If A, then B," the only definite conclusion we can draw is "If not B, then not A." In other words, if A always leads to B, and B doesn't happen, then A can't have happened either. The second sentence of the argument shows this principle: If not B (the lower-ranked player doesn't lose), then not A (there wasn't a fifth-set tiebreak). So we need to find another argument that follows this pattern: If A, then B; if not B, then not A. (A) CORRECT. If A (a woman with a family history of twins gets pregnant 3 times), then B (she will have 1 set of twins). Note that these numbers are precise: if she gets pregnant exactly three times, she will have exactly one set of twins. If not B (a woman with a family history of twins has 2 sets of twins - that is, not 1), then not A (she must have gotten pregnant either fewer than 3 times or more than 3 times - that is, not exactly 3 times). (B) If A (a salesman sells more product than anyone else), then B (he will earn an all-expenses-paid vacation). If B (Joe earned the trip), then A (he must have sold more than anyone else). We can see why logic rules do not include "if B, then A" as a logical conclusion: A may always lead to B, but B does not necessarily have to lead to A. There may be other ways to earn the trip besides selling more than anyone else. (C) If A (a newspaper's circulation surpasses 100,000), then B (the newspaper can charge a 50% premium). If not A (the circulation doesn't surpass 100,000), then not C (the newspaper cannot charge any premium). The final assertion here does not match the initial A / B argument We know nothing about any other premium the newspaper might charge; we are only given information about charging a 50% premium. (D) If A (a student is in the top 10% of the class), then B (she will earn a scholarship). If not A (Anna is not in the top 10%), then not B (she won't earn a scholarship). We can see why logic rules do not include "if not A, then not B" as a logical conclusion: A may always lead to B, but it doesn't have to be the only way to reach B. There may be other ways to earn a scholarship besides being in the top 10% of the class. (E) If A (the team wins the Super Bowl), then B (the players receive a bonus). If not A (a player was not on the winning team), then not B (the player won't receive a bonus). We can see why logic rules do not include "if not A, then not B" as a logical conclusion: A may always lead to B, but it doesn't have to be the only way to reach B. There may be other ways to earn a bonus besides winning the Super Bowl.

According to the National Science Foundation, in 2003 there were 198,113 female science and engineering graduate students, almost 42% of the graduate students in those fields, [twice as much as 1981]. a) twice as much as 1981 b) twice as many as 1981 c) double the figure for 1981 d) double what it was in 1981 e) a number double that of 1981's

Quantity/Comparisons: In the original sentence, "much" incorrectly references the quantity of female graduate students. Students are countable, so "many" is the correct term. Additionally, "as 1981" incorrectly completes the comparison, illogically comparing the number of people (the "198,113 female science and engineering graduate students") to a year ("1981"). (A) This choice is incorrect as it repeats the original sentence. (B) "As 1981" incorrectly completes the comparison, illogically comparing the number of people (the "198,113 female science and engineering graduate students") to a year ("1981"). (C) CORRECT. "Double the figure" places the emphasis on the number of female graduate students, and correctly completes the comparison between the number of people in one year (198,133 in 2003) and the number of people in another year (the figure for 1981). (D) "What it was" is wordy, awkward, and unclear. The singular pronoun "it" has no clear antecedent. (E) "Double that of 1981's" is wordy, awkward, and unclear. The singular pronoun "that" has no clear antecedent. The possessive "1981's" is not followed by a noun to possess.

The Action-Packed Gaming Company, based on the success of the previous season's video game featuring the character Sam Li, of the popular Fist of Awe series of martial arts movies, developed for the current season a similar martial arts game featuring a new character who is also a martial arts master. The new game had improved 3-D graphics, enhanced multiplayer capability, and dozens of new martial arts moves developed by real-life masters. However, marketing surveys showed that teenagers were uninterested in this new game, and the game sold very poorly. The passage implies that an explanation for the failure of the new game is based on doubt regarding which of the following assumptions? (A) Teenagers make purchasing decisions based on the technological merits of video games, not the name recognition of the games' main characters. (B) Buyers of video games prefer to purchase games based on popular movies. (C) The Fist of Awe series of movies was extremely popular with teenagers who regularly purchase video games. (D) Technological improvement from one video game to the next does not guarantee a corresponding increase in sales. (E) The successful video game may have benefited from advertising associated with the Fist of Awe movies, a benefit the failed video game did not have.

The argument can be summarized as follows: A video game based on a popular movie character was successful with teenagers. A company attempted to replicate this success with a similar game that had better technology, and a similar but unknown character. The new game was a failure with teenagers. It may be helpful to anticipate the argument's conclusion - that movie character Sam Li was a crucial factor in the first game's success - before proceeding to the question. The question asks us for the statement that is unlikely to be true - that is, the statement that is an assumption that, if untrue, would support an explanation for the new game's failure. A. This statement directly contradicts the facts of the passage. Teenagers preferred the less technologically advanced game featuring Sam Li to the more technologically advanced game featuring an unknown character. Therefore, doubt regarding the statement in answer choice A supports an explanation for the failure of the new game. Answer choice A is correct. B. This statement is supported by the facts of the passage. Incorrect. C. This statement supports the implied conclusion of the passage that Sam Li was a more important factor than improved technology. Incorrect. D. This statement is supported by the facts of the passage. Incorrect. E. This statement supports the implied conclusion of the passage that Sam Li was a more important factor than improved technology. Incorrect. The answer is A.

Letter to the editor: The Senate recently voted to keep the estate tax, a levy imposed on the transfer of property of the estate of a deceased person to a living person or organization. The primary argument in favor of the estate tax is that [it is a necessary component of a system of progressive taxation]. Not only is this argument faulty, but the [estate tax is also lousy for the economy]. Primarily, the estate tax directly penalizes savings and investment, two activities that are vital to create and sustain jobs. Furthermore, the estate tax undermines a strong work ethic by devaluing the link between work and reward. In the letter to the editor, the two sections in boldface play which of the following roles? a) The first is a fact in support of the author's position; the second is that position. b) The first is a consideration that the author argues is irrelevant; the second is a consideration in support of the author's argument. c) The first is an assumption that contradicts the author's position; the second is a premise in support of the author's position. d) The first is a conclusion that stands in opposition to the author's position; the second is the author's position. e) The first is a premise that clarifies the author's position; the second is a prediction that the author believes will hold true.

The author believes that the estate tax has a detrimental effect on the economy. This opinion, which is conveyed in the second boldface portion, stands in contrast to the opinion of estate tax proponents (that it is a necessary component of a progressive taxation system). (A) The first boldface portion is not a fact but an opinion cited by those in favor of the estate tax. The second boldface portion is, in fact, the author's position that the estate tax is lousy for the economy. (B) The author disagrees with the first boldface portion but does not argue that it is irrelevant. The second boldface portion is the author's position itself, rather than a consideration in support of this position. (C) The first boldface portion is not an assumption but a conclusion in favor of the estate tax. The second boldface portion is the author's position itself, rather than a premise in support of this position. (D) CORRECT. The first boldface portion is a conclusion in favor of the estate tax, a position that is in opposition to the author's position. The second boldface portion is the author's position that the estate tax is lousy for the economy. (E) The first boldface portion does not clarify the author's position; instead, it opposes the author's position. The second boldface portion is not a prediction but the author's conclusion concerning the estate tax.

In the 18th and 19th centuries, it was believed in many coastal American cities that the waterfront was an undesirable location for residential buildings. As a result, much of the waterfront in these cities was never developed aesthetically and instead was left to industry and commerce. Today, however, waterfront properties are generally seen as prestigious, as evidenced by the large sums paid for homes along the beach front. A developer who wishes to make a large profit would be wise to buy urban waterfront lots and erect residential buildings on them. Which of the following, if true, most strongly supports the claim made about urban waterfront properties? a) People today have more money, relatively speaking, to spend on real estate than they did in previous centuries. b) Homeowners will be willing to spend large sums on residential properties in traditionally industrial or commercial districts. c) Many urban waterfront lots are available for purchase. d) Many coastal American cities are encouraging developers to rehabilitate the waterfront through tax incentives. e) Properties in interior residential districts in coastal American cities are significantly more expensive than those along the waterfront.

The conclusion is that a developer who wishes to make a large profit would be wise to buy urban waterfront lots and erect residential buildings on them. The basis for that claim is that people pay large sums for beach front homes. We are asked to strengthen this argument. (A) This choice states that people have more buying power today than in previous centuries. This does not strengthen the claim that a developer will make money on urban waterfront properties. (B) CORRECT. This choice states that homeowners will be willing to spend large sums of money on residential properties in traditionally industrial or commercial districts. Since we know from the argument that urban waterfronts have traditionally been industrial, this fact strengthens the claim that a developer can make a profit on urban waterfront properties. (C) This choice states that many urban waterfront lots are available for purchase. This does not suggest, however, that a developer will be able to sell them after he or she builds on them. (D) This choice states that many coastal cities are giving tax breaks to developers who rehabilitate the waterfront. But this does not suggest that anyone will buy the developed properties. (E) This choice states that properties in the interior of cities are more expensive than those on the waterfront. Although waterfront properties are therefore cheaper to acquire, this does not necessarily mean that a developer can make a profit after buying such properties.

[The president's nominees to federal circuit courts have been judged conservative for their stands on hot-button issues.] But a review of their financial disclosure forms and Senate questionnaires reveals that the nominees are more notable for their close ties to corporate and economic interests, especially the energy and mining industries. Some of them were paid lobbyists for those same interests. Further, [the nominees with industry ties were overwhelmingly appointed to circuit courts regarded as traditional battlegrounds over litigation affecting these industries.] Independent observers who follow the federal bench believe that the extensive corporate involvement among so many of the nominees is unprecedented. In the argument above, the two portions in [boldface] pay which of the following roles? a) The first is a generalization that the author aims to attack; the second is that attack. b) The first is a pattern that the author acknowledges as true; the second is the author's conclusion based on that acknowledgment. c) The first is a phenomenon that the author accepts as true; the second is evidence in support of the author's conclusion. d) The first is the author's position based on the evidence cited; the second is a pattern presented in support of that position. e) The first is an exception to a rule introduced in the argument; the second provides the reasoning behind the exception.

The conclusion of the argument is that the nominees "are more notable for their close ties to corporate and economic interests" than for their positions on controversial issues. The first boldfaced statement is a recognition of the fact that the president's nominees have been branded conservative. The second boldfaced statement offers information in support of the assertion that the nominees are more notable for their corporate ties. So we need to find a choice that describes both statements accurately. (A) The author does not seek to attack the assertion made in the first statement. (B) The author does acknowledge the first statement as true. However, the second statement is not the conclusion. (C) CORRECT. The author does accept the first statement as true, and the second statement is indeed given in support of the conclusion. (D) The first statement is not the author's "position" (i.e., conclusion). (E) The first statement is not an exception to a rule, making the description of the second statement false as well.

According to psychologists, many dieters subconsciously prefer [a diet plan whose effectiveness is uncertain -- a consideration that, in case of failure, lets them blame the supposed ineffectiveness of the plan rather than their own lack of self-control.] a) a diet plan whose effectiveness is uncertain -- a consideration that, in case of failure, lets them blame the supposed ineffectiveness of the plan rather than their own lack of self-control b) an uncertain diet plan in terms of effectiveness: such plans allow them to believe that the supposed ineffectiveness of the plan is to blame, rather than that they lack self-control in case of failure c) diet plans with uncertain effectiveness, which will allow them to blame the supposed ineffectiveness of the plan, rather than to lack self-control, in case of failure d) uncertainly effective diet plans, allowing them to believe the plans that are supposedly ineffective, rather than that they lack self-control, in case of failure e) diet plans uncertain in effectiveness; in case of failure, allowing them to believe that the plan itself is ineffective rather than that they lack self-control

The content in the second part of this sentence -- describing what dieters will blame for the failure of a diet plan -- is a result of the dieters' uncertainty about the plan's effectiveness, not a result of the plan itself. Therefore, the sentence must be structured in a way that correctly indicates this relationship. Additionally, there should be proper parallelism between the two possible explanations given for the failure of the plan (i.e., the plan's ineffectiveness versus the dieters' lack of self-control). (A) CORRECT. "A diet plan whose effectiveness is uncertain" clearly and unambiguously indicates the nature of the uncertainty described: the effectiveness of the diet plan (not, say, the particulars of the plan itself) is uncertain. The following appositive modifier ("a consideration that...") is properly used to describe the entire preceding idea, namely, the idea that the diet plan's effectiveness is uncertain. The modifier "in case of failure" is set off by a comma from the entire parallel structure, properly indicating that the entire parallel structure (not just one of the parts) describes the situation in which the diet plan fails. Finally, the structure X rather than Y is formed with proper parallelism: both X ("the supposed ineffectiveness of the plan") and Y ("their own lack of self-control") are noun phrases. (B) The phrase "an uncertain diet plan in terms of effectiveness" is not only awkward, but also unclear: it seems to indicate that the diet plan itself, rather than the plan's effectiveness, is "uncertain." There is a lack of agreement between the plural "plans" (at the start of the second clause) and the singular "plan" (in the first clause); better agreement would be achieved if "such plans" were replaced with "such a plan." Finally, the modifier "in case of failure" is attached only to the second option in the parallel structure ("that they lack self-control"); this modifier should be placed so as to modify that entire parallel structure. (C) This choice places the two infinitives to blame... and to lack... in parallel, thus creating a nonsense meaning: the (illogical) implication that the diet plan "will allow them to blame" and "[will allow them] to lack self-control." This is incorrect; the dieters' second option is to blame a lack of self-control. (D) The phrase "uncertainly effective diet plans" is awkward and unclear. The comma + "allowing.." modifier should refer to the action of the preceding clause ("many dieters subconsciously prefer..."), but this meaning is nonsensical; it is not the case that, because the dieters prefer an "uncertainly effective" diet plan, this fact allows them to believe that the plan is ineffective. The two factors that could potentially be blamed are not written with proper parallelism: the first is a noun phrase ("the plans..."), while the second is an entire clause ("that they lack self-control"). Moreover, the first of these factors is written illogically: "to believe the plans" erroneously indicates that the dieters believe the plans themselves, when in fact the opposite is true, i.e., the dieters actually don't believe that the plan is effective. (E) In this choice, the portion following the semicolon ("allowing them to believe...") is a modifier, not a complete sentence. A semicolon must be followed by an independent clause (a complete sentence) containing both a subject and a verb.

Ethanol is a derivative of corn and other grains. When burned as fuel, it emits significantly lower levels of carbon monoxide, a major atmospheric pollutant, than does gasoline. For that reason, environmentalists claim that ethanol is a better source of energy than gasoline. Which of the following, if true, most strongly supports the environmentalists' claim? a) When burned as fuel, ethanol does not release any pollutants at higher levels than does gasoline. b) Ethanol is comparable in price to gasoline. c) Available supplies of corn are sufficient to supply several years' worth of ethanol. d) Most gasoline companies already possess the technology to produce ethanol. e) Ethanol can be used as heating fuel.

The environmentalists claim that ethanol is superior to gasoline because it emits lower levels of carbon monoxide, a known pollutant. This claim, however, assumes that ethanol does not release any other pollutants at levels greater than does gasoline. If ethanol released twenty times more sulfur into the environment than does gasoline, for example, perhaps it would be a less attractive alternative. (A) CORRECT. This establishes that ethanol is less polluting than gasoline. (B) The price of the fuels is not the issue in the argument. The environmentalists' claim is based on the levels of pollution that the fuels create. (C) Whether there is enough corn to produce ethanol is irrelevant to whether it is less polluting than gasoline. (D) Whether companies already possess the technology to produce ethanol is irrelevant to whether it is less polluting than gasoline. (E) Whether ethanol can be used as heating fuel is irrelevant to whether it is less polluting than gasoline.

Rust can deteriorate a steel pipe to such an extent that it [will snap easily, as though it was a twig.] a) will snap easily, as though it was a twig b) will snap easily, like a twig does c) will snap easily, as though it is a twig d) might snap easily, as though it was a twig e) will snap easily, as though it were a twig

The expression "as though" introduces a supposition that is contrary to fact (the pipe is not a twig). Such suppositions must be expressed in the subjunctive mood. For example, in the phrase "I wish I were rich," the verb "were" is in the subjunctive because the phrase expresses a desire contrary to fact. The appropriate singular form of the verb "to be" in the subjunctive mood is "were." (A) This choice is incorrect as it repeats the original sentence. (B) "Like" should not be used to introduce the comparative clause "a twig does." The word "as" should be used instead. "Like" is used to compare nouns only. (C) The verb "is" is not appropriate for the subjunctive mood. The appropriate singular form of the verb "to be" in the subjunctive mood is "were." (D) The verb "was" is not appropriate for the subjunctive mood. The appropriate singular form of the verb "to be" in the subjunctive mood is "were." In addition, this choice changes the meaning of the sentence. While the original sentence asserts that it is possible for rust to deteriorate a steel pipe to the point where it will snap easily, this choice asserts only that it is possible for rust to deteriorate the pipe to a point where it might snap, i.e., it implies a somewhat lesser degree of possible deterioration. (E) CORRECT. This choice correctly uses the subjunctive "were."

Disease, pollution, and overfishing have devastated the bountiful oyster harvests that once sustained the residents [of the Chesapeake Bay area.] a) of the Chesapeake Bay area b) in and around the Chesapeake Bay c) of the Chesapeake Bay d) around the vicinity of the Chesapeake Bay e) living in and around the Chesapeake Bay area

The given sentence is correct as written. "The residents of" a certain place is the proper idiom. It is also correct to refer to the residents living in the "area" of the Chesapeake Bay, rather than in the Bay itself. (A) CORRECT. The original sentence is correct as written. (B) This answer incorrectly implies that the residents are living "in" the Bay itself as well as the area surrounding the Bay. (Note that if we were talking about residents with houseboats or the like, they would be living "on" the Bay, not "in" it.) (C) This answer implies that the residents reside only in or on the Bay itself rather than near it or around the Bay area; though there may be some residents living on boats, the meaning of the original sentence indicates it was not intended to be limited to those living in or on the Bay. In addition, logic dictates that the residents cannot live "in" the Bay. (D) "Around the vicinity of" is both redundant and the incorrect idiom; to live in the "vicinity" of a landmark already includes the area "around" that landmark. The correct idiom is "in the vicinity of." (E) "Living in and around the Chesapeake Bay area" is redundant; living "in" a particular "area" implies living "around" that same area.

Though most people take it for granted now, the nationwide admission of students to colleges and universities based on academic merit is a relatively recent phenomenon, beginning only after World War II. a) Though most people take it for granted now, the nationwide admission of students to colleges and universities based on academic merit b) Though it is now taken for granted by most people, the admission of nationwide students to colleges and universities based on academic merit c) Now taken for granted by most people, colleges and universities admitting students based on their academic merit d) Most take them for granted now, but the admission of nationwide students to colleges and universities based on their academic merit e) Most people now take for granted that colleges and universities admit students nationally based on academic merit, and it

The original sentence begins with an opening modifier that correctly modifies the nationwide admission of students. (A) CORRECT. This choice is correct as it repeats the original sentence. (B) The use of the initial modifier in this choice is correct. However, the adjective "nationwide" is incorrectly applied to students, when it is meant to apply to the admission process. (C) Here, the modifier is adjacent to the subject "colleges and universities," incorrectly suggesting that colleges and universities are taken for granted as opposed to the admission process. (D) This sentence incorrectly uses the pronoun "them" to refer to the "admission" which is a singular subject. The use of the pronoun "their" is also unclear as the antecedent could be construed to be "colleges and universities" as opposed to the intended antecedent, "students." (E) Using the word "and" at the end of the underline makes the meaning of this sentence less clear by failing to draw an appropriate contrast between the current state of taking the nationwide admission of students for granted and the fact that it is a relatively recent phenomenon. A more appropriate word choice would be "but": "Most people now take for granted..., but it is a relatively recent phenomenon."

Though Frank Lloyd Wright is best remembered today [because of bold designs like the Guggenheim Museum] in New York City, most of his buildings were intended to blend into their surroundings. a) because of bold designs like the Guggenheim Museum b) for bold designs such as the one for the Guggenheim Museum c) because of bold designs such as the Guggenheim Museum d) because of bold designs such as that for the Guggenheim Museum e) for bold designs like the Guggenheim Museum's

The original sentence contains several errors. First, the phrase "remembered because of" is unidiomatic; the correct idiomatic construction is "remembered for." Second, "like" is incorrect because the Guggenheim is a specific example and the use of "such as" would be more appropriate to introduce examples. Third, the sentence incorrectly suggests that "the Guggenheim Museum" is a "design." More properly, the sentence should discuss the design "for" the museum. (A) This choice is incorrect as it repeats the original sentence. (B) CORRECT. "Because of" is replaced by "for." "Like" is replaced by "such as." And the use of "the one for" makes clear that the design is for the museum and is not the museum itself. (C) This choice does not correct "because of." Moreover, it incorrectly implies that the museum itself is a design. However, the use of "such as" in place of "like" is correct. (D) This choice does not correct "because of." Moreover, the relative pronoun "that" lacks a clear antecedent. It would be correct to say "The design for the house, like that for the factory, is beautiful," for example. But such a parallel structure does not exist in this sentence. However, the use of "such as" in place of "like" is correct. (E) This choice does not correct "like." Moreover, the possessive construction "Guggenheim Museum's" is awkward and unidiomatic. However, the use of "for" rather than "because of" is correct.

The bowerbirds of Australia derive their name from [the fact that the males build elaborate bowers of sticks and twigs to attract females, decorating them with flowers and other vegetation in a display of courtship.] a) the fact that the males build elaborate bowers of sticks and twigs to attract females, decorating them with flowers and other vegetation b) the elaborate bowers of sticks and twigs that the males build and decorate with flowers and other vegetation in order to attract females c) the elaborate bowers of sticks and twigs, decorated with flowers and other vegetation that the males use to attract females d) the fact that the males build elaborate bowers of sticks and twigs, having decorated them with flowers and other vegetation, to attract females e) the elaborate bowers of sticks and twigs that are built by the males and decorated with flowers and other vegetation to attract females

The original sentence contains the pronoun "them" but it is not grammatically clear whether the pronoun's antecedent is "bowers of sticks and twigs" or "females." Logically, we know that the antecedent is "bowers", so we need to find a replacement that makes this clear. Moreover, the bowerbird does not derive its name from the fact that it builds bowers, but from the bowers themselves. (A) This choice is incorrect as it is the same as the original sentence. (B) CORRECT. This choice rewrites the sentence to make it clear that the name derives from the bowers and not from the fact of building them, and it also eliminates the pronoun "them" and instead refers to "structures" to make the relationship clear. (C) This choice does not make it clear that the males build the bowers and decorate them. Instead, it seems to suggest that the bowers exist on their own and that the male uses only the flowers and vegetation to attract females. (D) This choice uses the phrase "having decorated them" improperly. It is not necessary to use "having" in this context because the sentence describes an ongoing event, not one that occurred in the past. (E) This choice is in the passive voice, which is not preferable to active voice when a grammatical active version (such as B) is also offered. Moreover, the choice implies that the males only build the bowers. Since the original sentence clearly indicates that the males also decorate the bowers, this choice changes the meaning unacceptably.

[Like many of his contemporaries did], Bob Dylan wrote songs that became anthems for a generation of antiwar activists. a) Like many of his contemporaries did, Bob Dylan wrote songs b) Bob Dylan wrote songs like many of his contemporaries c) As did many of his contemporaries, Bob Dylan wrote songs d) Like the songs of his contemporaries, Bob Dylan wrote songs e) As did many of Bob Dylan's contemporaries, he wrote songs

The original sentence incorrectly uses "Like" to compare two clauses: "Like many of his contemporaries did, Bob Dylan wrote songs." "Like" can be used to compare nouns, but not phrases containing verbs (clauses). (A) This choice is incorrect as it repeats the original sentence. (B) As used in this sentence, "like" seems to compare the noun "songs" with the noun "contemporaries," implying that Bob Dylan's "songs" were similar to his "contemporaries." This comparison is illogical, as songs cannot be compared with people. (C) CORRECT. "As" is correctly used to compare two phrases containing verbs. (D) The modifying phrase "Like the songs of his contemporaries" incorrectly modifies the adjacent noun "Bob Dylan," implying that Bob Dylan is similar to the songs of his contemporaries. Songs cannot be logically compared with people. (E) While "as" is correctly used to compare two phrases containing verbs, the subject pronoun "he" incorrectly refers back to the possessive noun "Bob Dylan's." Only possessive pronouns can be used to refer to possessive nouns.

During the past decade, the labor market in France has not been operating according to free market [principles, but instead stifling functioning through its various government regulations restricting the hiring and firing of workers.] a) principles, but instead stifling functioning through its various government regulations restricting the hiring and firing of workers b) principles, instead it has been functioning in a stifled manner as a result of various government regulations that restrict the hiring and firing of workers c) principles, rather functioning despite being stifled as a result of government regulations that variously restrict worker hiring and firing d) principles; the hiring and firing of workers is restricted there by various government regulations, its functioning being stifled e) principles; instead, its functioning has been stifled by various government regulations restricting the hiring and firing of workers

The original sentence is problematic in its use of the possessive pronoun "its." The antecedent to "its" is the "labor market," which incorrectly and illogically suggests that the labor market is somehow possessing or passing government regulations itself. In addition, the original sentence incorrectly uses active rather than passive voice to describe the effects imposed on the "labor market" by government regulations, thus illogically suggesting that the "labor market" itself is stifling functioning, rather than being stifled by other forces. (A) This choice is incorrect as it repeats the original sentence. (B) This choice incorrectly uses a comma to connect two independent clauses, thus creating a run-on sentence. Two independent clauses must be connected either by a conjunction, such as "and" or "but," or by a semicolon. (C) This choice incorrectly uses "stifled" to modify the labor market itself, as opposed to its functioning. Also, "variously restrict" is awkward; various is used more appropriately to modify "government regulations," rather than the manner in which the regulations restrict worker hiring and firing. (D) In order to properly use a semicolon, both the clause before and after the semicolon must be independent clauses or sentences, and the clauses must be closely related in meaning. In this choice, the underlined portion, though grammatically correct, does not stand alone as an effective independent clause. Also, the pronoun "its" lacks a clear antecedent. (E) CORRECT. This answer choice correctly uses the semicolon to connect two independent but closely related clauses. In addition, the pronoun "its" clearly and unambiguously refers to the "labor market."

According to the international investment memorandum recently signed in Geneva, France is one of the 4 European nations [planning to provide fewer] tax incentives for foreign investment in production of heavy industrial machinery. a) planning to provide fewer b) planning to provide less c) planning on providing fewer d) which is planning on providing fewer e) that is planning to provide less

The original sentence uses the correct idiomatic construction "planning to provide." Additionally, the appropriate quantity modifier "fewer" is used to refer to the countable noun "incentives." (A) CORRECT. This answer choice is correct as it repeats the original sentence. (B) This answer choice uses the incorrect quantity modifier "less" to refer to the countable noun "incentives." The modifier "less" can be applied only to non-countable nouns, for example "less water" or "less evidence." Countable nouns, such as "incentives," should be described by the modifier "fewer" rather than "less." (C) This answer choice uses the unidiomatic construction "to plan on." The appropriate idiom is "to plan to." (D) This answer choice also uses the less idiomatic construction "planning on." The singular verb "is planning" does not agree with the plural subject "nations." Additionally, the use of "which" is incorrect in this context. "Which" should only be used to introduce a modifier that is not integral to the meaning or structure of the sentence; here, the fact that France is one of the 4 nations planning to provide tax incentives is the main purpose of the sentence, and the omission of the language following "which" would result in an incomplete sentence. (E) This answer choice supplies the incorrect singular verb construction "is planning" that does not agree with the plural subject "nations." (as "nations" is followed by the correct relative pronoun, "that", the verb must agree with the plural noun immediately preceding "that"). Furthermore, this answer choice uses the incorrect quantity modifier "less" to refer to the countable noun "incentives." The modifier "less" can be applied only to non-countable nouns, for example "less water" or "less evidence." Countable nouns, such as "incentives," should be described by the modifier "fewer" rather than "less."

According to a recent magazine article, of those office employees who typically work 8 hours at the office each day but sometimes say that they will work at home on a particular day, 25 percent actually work less than one hour. At the same time, over 90 percent of those same office employees believe they are more productive working at home than working in their office. The statements above, if true, best support which of the following conclusions about the office employees discussed in the article? (A) On average, the office employees working at home for a day work fewer hours than office employees working at the office. (B) 10 percent of the office employees are less productive working from home than working in their office. (C) At least 15 percent of the office employees do not define productivity exclusively in terms of the number of hours worked. (D) At least 25 percent of the office employees can complete the same amount of work in one hour at home as in 8 hours at the office. (E) Some of the office employees make statements regarding their productivity that are not in fact true.

The passage presents information about what office employees who work 8-hour days and who have worked at home told a certain magazine. The first piece of information is about what some of those office employees actually do: 25 percent of office employees actually work less than an hour on days that they work at home. The second piece of information is about what some of those office employees believe: 90 percent believe that they are more productive working at home than at the office. A proper GMAT conclusion must be provable by those two pieces of information. (A) The passage only provides information about the working hours of 25 percent of the office employees. The passage does not provide any information regarding the working hours of the other 75 percent, hence, it is not possible to conclude anything about the office employees on average. For example, it is possible that the other 75 percent of the office employees work 14 hour days when working from home. It is also possible that they work 6 hour days when working from home. (B) The passage provides no information about the actual productivity of any of the office employees. It only provides information about what the office employees believe about their productivity. (C) CORRECT. 90 percent of the office employees believe that they are more productive at home than at work. At the same time, 25 percent of the office employees actually work fewer hours when they work at home than when they work at the office. The overlap between these two groups is at least 15 percent of all of the office employees. This group of employees believes that they are more productive at home than at work and yet this group actually works fewer hours at home than at work. Thus, these employees must not define productivity exclusively in terms of the number of hours worked. (D) The passage discusses the actual work hours of 25 percent of the office employees. Then it describes the beliefs of 90 percent of office employees regarding their productivity. First, there is no necessary link between an individual's beliefs about his or her productivity and that individual's actual productivity; hence, no conclusion can be made regarding actual productivity from the information about beliefs. Second, the number of hours worked alone is not an indication of productivity; it is possible, for example, that an employee who works 1 hour is more productive in terms of work done per hour than when he works 8 hours and yet that employee might still accomplish more total work when working 8 hours. Therefore, it is not possible to conclude anything regarding productivity for any of the office employees. (E) The fact that 90 percent of the office employees believe they are more productive at home than at work does not necessarily contradict the fact that 25 percent of the office employees work fewer hours at home than at work. It is possible to work fewer hours and still be more productive.

Independent contractors pay higher [taxes and paid less consistently than are statutory employees, but they, unlike statutory employees, are freely allowed to] perform the same type of work for multiple businesses. a) taxes and paid less consistently than are statutory employees, but they, unlike statutory employees, are freely allowed to b) taxes and are paid less consistently than statutory employees; unlike statutory employees, though, independent contractors are free to c) taxes, are paid less consistently than statutory employees, and, unlike statutory employees, they may freely d) taxes and are paid less consistently than are statutory employees; in addition, unlike statutory employees, independent contractors can freely e) taxes and paid less consistently than are statutory employees; moreover, unlike statutory employees, they are allowed to

The sentence describes several differences between the situation of independent contractors and that of statutory employees. The first two observations depict the situation of independent contractors as less favorable than that of statutory employees, but the reverse is true of the final observation; therefore, the sentence must properly express the contrast between the first two observations and the last one. Additionally, the first two observations—each of which describes a relative disadvantage faced by independent contractors—should be written in parallel. (A) And links the constructions pay... and paid... These constructions are not parallel: the first is a verb phrase, but the second is a participial modifier. The helping verb to be (appearing here as are) cannot be used to stand for an action verb, so the construction are statutory employees is not parallel to pay higher taxes. The adverb freely is incorrectly placed; it is intended to describe the act of performing similar work for other companies, but, in its current location, it illogically modifies "allowed". Finally, the modifier unlike statutory employees is used to modify a pronoun (they) rather than a noun, resulting in an awkward and unacceptable construction. (B) CORRECT. The two disadvantages faced by independent contractors are properly represented by two parallel verbs (pay... and are paid...), and the transition though is appropriately used to describe the contrast between the first two observations and the last. (C) This construction contains a list of three constructions (pay..., are paid..., and they may freely...) that is incorrect in terms of both meaning and grammar. The construction x, y, and z makes no sense here, since the last observation contrasts with the first two; the same list is also grammatically nonparallel, since pay and paid are verbs but they may... is a full clause with a subject and a verb. (D) The transition in addition implies that the following statement reinforces the idea(s) stated earlier; in this sentence, the last observation contrasts with the first two, so in addition is inappropriate. (E) And links the constructions pay... and paid... These constructions are not parallel: the first is a verb phrase, but the second is a participial modifier. The helping verb to be (appearing here as are) cannot be used to stand for an action verb, so the construction are statutory employees is not parallel to pay higher taxes. Finally, the transition moreover implies that the following statement reinforces the idea(s) stated earlier; in this sentence, the last observation contrasts with the first two, so moreover is inappropriate.

During the twentieth century, the study of the large-scale structure of the universe evolved from the theoretical to the practical; the field of physical cosmology was made possible [because of both Einstein's theory of relativity and] the better ability to observe extremely distant astronomical objects. (A) because of both Einstein's theory of relativity and (B) by both Einstein's theory of relativity and (C) by Einstein's theory of relativity and also (D) because of Einstein's theory of relativity and also (E) as a result of both Einstein's theory of relativity and

The underlined portion of the sentence introduces two idioms: made possible by and both X and Y. The former idiom is incorrectly presented in the sentence as made possible because of. (A) The choice is incorrect as it repeats the original sentence. (B) CORRECT. This choice uses both idioms correctly: made possible by and both X and Y. (C) This choice corrects the first idiom (made possible by) but introduces a new error by removing both and replacing it with and also, which is redundant. (D) This choice repeats the original idiom error made possible because of. It also introduces a new error by removing both and replacing it with and also, which is redundant. (E) This choice presents the incorrect idiom (made possible as a result of) rather than the correct idiom, made possible by.

[The reason that certain spicy foods, such as the Habanero pepper, makes some people sweat is because they contain a chemical that stimulates the same nerve endings in the mouth as does a rise in temperature]; this stimulation sometimes results in the activation of certain biological cooling mechanisms, one of which is perspiration. a) The reason that certain spicy foods, such as the Habanero pepper, makes some people sweat is because they contain a chemical that stimulates the same nerve endings in the mouth as does a rise in temperature b) The reason that certain spicy foods, such as the Habanero pepper, makes some people sweat is that these foods contain a chemical that stimulates the same nerves in the mouth like a rise in temperature c) Certain spicy foods, such as the Habanero pepper, make some people sweat because they contain a chemical that stimulates the same nerves in the mouth like a rise in temperature does d) The reason that certain spicy foods, such as the Habanero pepper, make some people sweat is that these foods contain a chemical that stimulates the same nerves in the mouth as does a rise in temperature e) The reason that certain spicy foods, such as the Habanero pepper, make some people sweat is because they contain a chemical that stimulates the same nerves in the mouth as does a rise in temperature

There are three errors in the sentence. First, the plural subject "foods" does not agree with the singular verb "makes." Second, the pronoun "they" has an ambiguous referent: it could refer either to "foods" or "people." Third, "the reason X is because Y" is redundant. (A) This choice is incorrect as it repeats the original sentence. (B) The plural subject "foods" does not agree with the singular verb "makes." In addition, it is not clear to what "a rise in temperature" is being compared; a clearer and more logical comparison is "a chemical that stimulates ... as does a rise in temperature." (C) The pronoun "they" has an ambiguous referent: it could refer either to "foods" or "people." In addition, the clause "a rise in temperature does" should be introduced by "as" rather than "like," which, in this context, should be used to introduce a noun. The correct forms of the idiom are "X behaves like Y," "X behaves as Y does", or "X behaves as does Y." (D) CORRECT. The choice corrects all three errors in the original sentence. The plural "foods" agrees with the plural "make." The ambiguous "they" is replaced by "these foods," and the redundant construction "the reason X is because Y" is gone. (E) The pronoun "they" has an ambiguous referent: it could refer either to "foods" or "people." In addition, the clause "the reason X is because Y" is redundant. The correct forms of the idiom are "X is because Y" and "the reason X is Y."

The amount of money estimated to be lost by Hollywood studios due to DVD piracy overseas increased to more than $500 million last year. As a consequence, Hollywood studios have little prospect of making money this year by exporting their films worldwide. All of the following, if true, weaken the argument EXCEPT: a) $500 million represents a very small fraction of overseas profits. b) New laws and heightened international enforcement aimed at this piracy were implemented in January of this year and have been effective. c) Even in countries where DVD piracy is widespread, theatrical first-run distribution is both very popular and profitable. d) Last year, all films that were nominated for an Academy Award were known to have been pirated on DVD. e) Domestic DVD sales were sharply higher last year; in the past, strong domestic sales have been an indicator of higher international DVD sales in the following year.

This argument concludes that "Hollywood studios have little chance of making money this year by exporting their films worldwide," based on the premise that profits from piracy overseas rose sharply last year. The argument assumes that no other relevant conditions have changed for the better since then. The question asks for a choice that does not weaken the argument, which means that the correct answer will either strengthen it or is irrelevant. (A) This choice weakens the argument. It attacks the necessary assumption that $500 million is a large enough amount to destroy Hollywood profits. (B) This choice weakens the argument. It attacks the necessary assumption that there would not be a heightened and effective law enforcement campaign against piracy. (C) This choice weakens the argument. It attacks the assumption that there are no other ways for Hollywood studios to profit from export of its films than DVDs. (D) CORRECT. This choice is irrelevant. The conclusion (and premise) concern money. That all of the Academy Award-nominated films were pirated last year does not increase or diminish the probability of Hollywood studios making money this year. Thus, this does not weaken the argument. (E) This choice makes the conclusion less likely by providing a premise that suggests that this year's profits internationally will rise.

To prevent overcrowding, last month the town zoning board limited the number of new buildings that can be constructed in the town in any given year. The board claims that doing so will preserve open spaces and lessen the strain on municipal resources such as schools and garbage disposal. Critics of the changes argue that the plan will harm the community or, at the very least, will fail in its purpose. Which of the following most supports the claims of the critics of the plan? a) Other towns have had mixed success with similar zoning plans. b) No new schools have been built in the town in ten years. c) Property taxes in the town are higher than in neighboring towns. d) Under the new plan, developers may still erect apartment buildings. e) The nearest garbage dump is several miles away from the town.

This argument discusses a plan with the stated goal "to prevent overcrowding." Two points of view are represented in the argument: one is that of the town zoning board, the originators of the plan, and the other is that of critics of the plan. The question asks for information that would most support the claims of the critics of the plan. Conversely, the correct answer would most weaken the logic of the town zoning board. (A) How other towns fared under similar zoning plans is irrelevant to this argument, unless additional information were presented to connect the situation of this town to those. We have no such connecting information, so this choice neither strengthens nor weakens either point of view. (B) How long it has been since the construction of the last school in town is irrelevant to this argument. The argument addresses methods to prevent overcrowding, an issue that is not directly related to school construction. (C) The argument never mentions property taxes, so how property taxes in this town compare to those in neighboring towns is irrelevant. (D) CORRECT. The argument states that "the town zoning board limited the number of new buildings that can be constructed in the town in any given year." The goal of this plan is to prevent overcrowding, but only does so indirectly: the town zoning board plan limits the number of new buildings, not the number of new town residents. Either an apartment building or a single family house would only count as one new building, yet the apartment building could draw many more new residents to the town than would the single family house. This plan might not be the best way to achieve the stated goal, and thus this statement supports the claims of the critics of the plan. (E) The distance to the nearest garbage dump is irrelevant to an argument about how to prevent overcrowding.

According to a recent study on financial roles, one-third of high school seniors say that they have "significant financial responsibilities." These responsibilities include, but are not limited to, contributing to food, shelter, or clothing for themselves or their families. At the same time, a second study demonstrates that a crisis in money management exists for high school students. According to this study, 80% of high school seniors have never taken a personal finance class even though the same percentage of seniors has opened bank accounts and one-third of these account holders has bounced a check. Which of the following conclusions can be properly drawn from the statements above? a) High schools would be wise to incorporate personal finance classes into their core curricula. b) At least one-third of high school seniors work part-time jobs after school. c) The number of high school seniors with significant financial responsibilities is greater than the number of seniors who have bounced a check. d) Any high school seniors who contribute to food, shelter, or clothing for themselves or their families have significant financial responsibilities. e) The majority of high school students have no financial responsibilities to their families.

This passage relates information from two studies concerning high school seniors: the first discusses the financial responsibilities of high school seniors, while the second explains the coursework in finance taken by typical high school seniors. On the GMAT, a proper response to a draw-a-conclusion question must be directly supported by evidence from the passage. (A) Although it might be true that schools would be wise to educate students in finance, this is an opinion; it doesn't necessarily need to follow from the given evidence. A conclusion must be directly supported by evidence from the passage without any additional information or assumptions. (B) The fact that one-third of high school seniors claim "significant financial responsibilities" to their families does not necessarily mean that these same students work "part-time jobs after school." There are many possible ways that these students might earn money for their families. If they do work, they might work on weekends or over the summer, for example. (C) CORRECT. The first study states that one-third of all high school seniors have significant financial responsibilities to their families. The second study states that 80% of seniors have opened a bank account, and of this 80%, one-third has bounced a check. The number of seniors that has bounced a check (one-third of 80%) is fewer than the number of seniors with significant financial responsibilities to their families (one-third of 100%). (D) The passage states that certain high school seniors who contribute to the food, shelter, or clothing for themselves or their families rate themselves as having significant financial responsibilities. This does not mean that any high school senior who contributes to these categories has significant financial responsibilities. (E) The passage states that one-third of high school seniors say that they have "significant financial responsibilities." This in no way indicates that the other two-thirds have "no" responsibilities. Because no information is given about the other two-thirds of the students, a reasoned conclusion cannot be drawn about them.

Most cable television companies currently require customers to subscribe to packages of channels, but consumer groups have recently proposed legislation that would force the companies to offer a la carte pricing as well. Subscribers would pay less, argue the consumer groups, because they could purchase only the desired channels. However, the cable industry argues that under the current package pricing, popular channels subsidize less-popular ones, providing more options for viewers. For this reason, the industry claims that it is always cheaper for the consumer to purchase many bundled channels than to buy them individually. Which of the following would be most important for the government to determine before deciding whether to require cable television companies to offer a la carte pricing in order to reduce consumer costs? a) Whether the total number of channels offered to consumers would decrease, along with programming diversity, as a result of the a la carte pricing structure b) Whether advertising revenue for the cable television companies would decrease as a result of the a la carte pricing structure c) Whether a large number of consumers would, in fact, significantly reduce the number of channels purchased if given the option of purchasing them individually d) Whether the number of cable television consumers has been declining as a result of new avenues of content delivery, such as the Internet e) Whether a la carte subscribers would be required to have new television set-top boxes

Two points of view are expressed in the argument: consumer groups claim that a la carte pricing will reduce consumer costs, while the cable television industry claims that the current package pricing structure is most cost effective for consumers. If the goal of the government is to reduce the cost of cable television for consumers, it is critical for the government to determine whether adding the option of a la carte pricing is likely to save consumers money. (A) According to the argument, the government's decision is to be based only on the costs to consumers, not the number of channels available to them. If there were some pricing consequences as a result of this loss of diversity, the point might be relevant, but we are given no such information. (B) According to the argument, the government's decision is based only on the costs to consumers, not the advertising profits of the cable television companies. (C) CORRECT. If consumers would not choose to order all of the channels they currently buy as part of a package subscription, then the television industries' claim that a la carte pricing would always be more expensive is suspect. If many consumers only watch and wish to pay for a few of their favorite channels, a la carte pricing could very well result in lower cable bills for those consumers. (D) According to the argument, the government's decision is to be based on the costs to consumers, not the number of consumers who subscribe. If there were some pricing consequences as a result of a loss of subscribers, the point might be relevant, but we are given no such information. (E) According to the argument, the government's decision is to be based only on the costs to consumers, not the technical equipment requirements a change in cost structure would require.


Conjuntos de estudio relacionados

1 - Accounting: The Language of Business

View Set

Network+ Chapter 5 Networking Devices

View Set

Personal property, Real property, Bailments, and Landlord-Tenant Law

View Set

Religion-Understanding the Scriptures Chapter 3

View Set